Đến nội dung

IHateMath

IHateMath

Đăng ký: 10-02-2016
Offline Đăng nhập: Riêng tư
***--

#687351 Đề luyện tập olympic khối 10 VMF lần 2 tháng 7

Gửi bởi IHateMath trong 12-07-2017 - 21:31

Bài 5.

Đáp số đúng là $80$ người. 

 

Người $1$ cho đến người $8$: Chỉ không đồng tình với các cải cách lần thứ $1,2,3$.

Người $9$ cho đến người $16$: Chỉ Không đồng tình với các cải cách lần thứ $1,2,4$.

Người $17$ cho đến người $24$: Chỉ Không đồng tình với các cải cách lần thứ $1,2,5$.

Người $25$ cho đến người $32$: Chỉ Không đồng tình với các cải cách lần thứ $1,3,4$.

Người $33$ cho đến người $40$: Chỉ Không đồng tình với các cải cách lần thứ $1,3,5$.

Người $41$ cho đến người $48$: Chỉ Không đồng tình với các cải cách lần thứ $1,4,5$.

Người $49$ cho đến người $56$: Chỉ Không đồng tình với các cải cách lần thứ $2,3,4$.

Người $57$ cho đến người $64$: Chỉ Không đồng tình với các cải cách lần thứ $2,3,5$.

Người $65$ cho đến người $72$: Chỉ Không đồng tình với các cải cách lần thứ $2,4,5$.

Người $73$ cho đến người $80$: Chỉ Không đồng tình với các cải cách lần thứ $3,4,5$.

Người $81$ cho đến người $96$: Đồng tình với tất cả các cải cách.

Rõ ràng, với mỗi cải cách, có đúng $8\cdot 6=48$ người không đồng tình.




#686955 Có 10 bậc cầu thang

Gửi bởi IHateMath trong 08-07-2017 - 15:19

Ta tổng quát bài toán lên thành $n$. Gọi $s_n$ là số cách đi. Để ý rằng các cách đi có thể được xếp vào $n$ nhóm dựa vào số bậc trong bước đi cuối cùng:

- Bước $1$ bước duy nhất: $1$ cách duy nhất

- Bước cuối cùng có $n-1$ bậc: $s_1=1$ cách

.

.

.

- Bước cuối cùng có $1$ bậc: $s_n$ cách

Vậy $s_n=s_{n-1}+\dots +s_1+1=s_{n-1}+s_{n-1}=2s_{n-1}=2^{n-1}s_1=2^{n-1}$. 




#686862 Tìm STN n lớn nhất thỏa mãn 2017! $\vdots$5$^{n...

Gửi bởi IHateMath trong 07-07-2017 - 21:05

Nếu không nhầm thì:

n=$\left [\frac{2017}{5} \right ]+\left [\frac{2017}{5^{2}} \right ]+\left [\frac{2017}{5^{3}} \right ]+\left [\frac{2017}{5^4} \right ]$

Chỗ này thực chất là công thức Legrendre: https://en.wikipedia...endre's_formula




#686861 Bài toán đếm trong Olympic 27-4 BRVT

Gửi bởi IHateMath trong 07-07-2017 - 20:58

Bài toán đã được giải tại đây: https://diendantoanh...án-11/?p=673660




#686806 Cho $n$ là số tự nhiên khác $0$, $a$ là ước ngu...

Gửi bởi IHateMath trong 07-07-2017 - 16:12

Chỉ đơn thuần là nhân hai vế với $\left( \frac{k}{n}\right) ^2$ thôi, còn $k$ đã là số nguyên dương thì $k^2+2k$ cũng là số nguyên dương.


  • tcm yêu thích


#686787 Cấp số cộng và phân hoạch tập $Z^+$

Gửi bởi IHateMath trong 07-07-2017 - 15:02

Cho số nguyên tố $p\geq 3$ và dãy số nguyên phân biệt $\{ a_i\}_{i=1}^p$. Chứng minh rằng nếu tập số nguyên dương $Z^+$ có thể phân hoạch thành các tập $A_1,A_2,\dots ,A_p$ sao cho với các $i\in\{ 1,2,\dots ,p\}$ thì các tập $A_i+a_i=\{x+a_i|x\in A_i\}$ là đồng nhất thì dãy $\{ a_i\}$ phải lập thành một cấp số cộng.




#686718 Tô màu được nhiều nhất bao nhiêu ô trên bàn cờ?

Gửi bởi IHateMath trong 06-07-2017 - 17:59

Kí hiệu $f(n)$ thay cho số lớn nhất các ô vuông được tô màu.

Đáp số: $f(n)=\frac{n^2}{2}$ nếu $n$ chẵn và $f(n)=\frac{n(n+1)}{2}$ nếu $n$ lẻ.

Chứng minh

Trường hợp $n$ chẵn là hiển nhiên. Ta chỉ xét trường hợp $n$ lẻ. Ta sẽ chứng minh bằng quy nạp. Trong trường hợp cơ sở $n=1$ hiển nhiên $f(1)=1$, đúng. Giả sử ta đã chứng minh được rằng $f(2k-1)=(2k-1)k$ với $k$ nguyên dương nào đó. Trước hết ta sẽ chứng minh

$$f(2k+1)\leq(2k+1)(k+1)\quad (1)$$ 

Thật vậy ta cắt bảng $(2k+1)\times (2k+1)$ thành các bảng con như sau: một bảng $(2k-1)\times (2k-1)$ sao cho nó có chung một đỉnh với bảng lớn và phần còn lại cắt thành các bảng con $2\times 2$ chừa ra một hình L - Tetramino (là hình gồm 4 ô vuông xếp với nhau tạo thành hình chữ "L").

Theo giả thuyết quy nạp, trên bảng con $(2k-1)\times (2k-1)$ tô được nhiều nhất $(2k-1)k$ ô vuông còn trên hình L - Tetramino tô được nhiều nhất $3$ ô (điều này khá hiển nhiên), các bảng con $2\times 2$ (có đúng $2k-1$ bảng như vậy) thì tô được nhiều nhất $2$ ô, nên trên bảng ban đầu ta tô được nhiều nhất:

$$(2k-1)k+2(2k-1)+3=(2k+1)(k+1).$$

Điều này chứng tỏ $(1)$ đúng.

Giờ ta sẽ chỉ ra rằng dấu "=" trong $(1)$ là có thể xảy ra. Phần này thì khá đơn giản, chỉ cần tô xen kẽ các hàng là xong.

Vậy công thức đã cho cũng đúng với $n=2k+1$. Chứng minh kết thúc.

Bình luận: Trong trường hợp $n=5$ bài toán từng xuất hiện trên tạp chí THTT.




#684860 Cho 3 số dương $a,b,c$ thoả mãn: $\sum \frac{1...

Gửi bởi IHateMath trong 18-06-2017 - 00:49

Bài này tương tự bài toán $\text{A2 IMO 2009 Shortlist}$, chỉ thay điều kiện $\frac{1}{a}+\frac{1}{b}+\frac{1}{c}=a+b+c$. Lời giải cũng tương tự như bài toán nói trên.




#684673 $(a+b+c)^3\geq 6\sqrt{3}(a-b)(b-c)(c-a)$

Gửi bởi IHateMath trong 16-06-2017 - 10:55

Không mất tính tổng quát, giả sử $a\geq b,\, a\geq c$. Ta xét hai trường hợp sau:

$\bullet$ Trường hợp $1$: $b\geq c$. Khi đó $(a-b)(b-c)(c-a)\leq0$, trong khi $a+b+c\geq 0$. Bất đẳng thức đa cho đúng. Dấu "=" xảy ra khi $a=b=c=0$.

$\bullet$ Trường hợp $2$: $b<c$. Khi đó, do $b\geq 0$ nên ta có $$(a+b+c)^3\geq (a+c)^3,\, (a-b)(b-c)(c-a)=(a-b)(c-b)(a-c)\leq ac(a-c).$$

Như vậy ta chỉ còn cần phải chứng minh

$$(a+c)^3\geq 6\sqrt{3} ac(a-c).$$

Đặt $t=\frac{a}{c}$ thì $t\geq 1$ và bất đẳng thức trên trở thành

$$(t+1)^3\geq 6\sqrt{3} t(t-1)\iff (t-2-\sqrt{3})^2(t+7-4\sqrt{3})\geq 0,$$

hiển nhiên đúng. Dấu "=" xảy ra khi $b=0,\, a=(2+\sqrt{3})c\, (c>0)$.

 




#684581 Không hiểu một vài chỗ trong bài chứng minh sự tồn tại vô hạn các số nguyên tố.

Gửi bởi IHateMath trong 14-06-2017 - 23:55

Cảm ơn bạn nhiều nhé.

 

Mình có 1 bài này nữa cũng thắc mắc, mong bạn giải đáp thêm:

 

Cho số nguyên $n$ là hợp số, $n > 1$. Chứng minh rằng $n$ có ước nguyên tố $p \leqslant \sqrt{n}$.

 

Cách giải (của sách):

 

Do $n$ là hợp số nên $n$ có thể viết dưới dạng $n = a.b$ với $a, b \in N, a > 1, b > 1$. Bây giờ nếu cả $a > \sqrt{n}$ và $b > \sqrt{n}$ thì $ab > \sqrt{n}.\sqrt{n} = n$, mâu thuẫn. Do đó phải có $a \leqslant \sqrt{n}$ hoặc $b \leqslant \sqrt{n}$, và do đó $n$ có ước nguyên tố $p \leqslant \sqrt{n}$. Bài toán được chứng minh.

 

1. Vì sao $n$ là hợp số nên $n$ có thể viết được dưới dạng $n = a.b$ với $a, b \in N, a > 1, b > 1$? (giả sử $n = 4$ thì trường hợp $4 = 4.1$ cũng có thể xảy ra mà?)

2. Vì sao có $a \leqslant \sqrt{n}$ hoặc $b \leqslant \sqrt{n}$ mà $n$ lại có ước nguyên tố $p \leqslant \sqrt{n}$?

 

Mình cảm ơn.

Em thấy lời giải này đâu có sai sót nhỉ?

Để mình giải thích cho bạn.

1. $n$ là hợp số thì $n$ phải có ước số khác $1$ và $n$, ta gọi nó là $d$ đi, thì chả phải $\frac{n}{d}$ và $d$ đều lớn hơn $1$ hay sao? Bạn đưa ra ví dụ $4=4\cdot 1$ để làm gì nhỉ, vì rõ ràng $4=2\cdot 2$.

2. Giả sử $a\leq \sqrt{n}$ thì $a$ chắc chắn sẽ có ước nguyên tố và ta gọi nó là $p$, thì $n$ chia hết cho $p$, mà $p\leq a\leq \sqrt{n}$ nên sẽ có điều cần chứng minh.


  • tcm yêu thích


#684149 Đề tuyển sinh vào lớp 10 THPT chuyên Lê Quý Đôn - Quảng Trị năm học 2017-2018

Gửi bởi IHateMath trong 12-06-2017 - 00:03

Câu 6.

 

Ta quy về chứng minh Bài Toán sau: Cho tam giác $ABC$ nhọn. $M$ là trung điểm của $BC$. Đường cao $BH , CK$ lần lượt cắt đường thẳng vuông góc với $AM$ tại $E , F$. Chứng minh rằng: $AE=AF$

 

Mình có cách giải khác khác cho bài toán này. Ta cần chứng minh $$\frac{AK}{\cos\angle BAF}=\frac{AH}{\cos\angle CAE}\iff \frac{\cos\angle BAF}{\cos\angle CAE}=\frac{AK}{AH}.$$

Do các góc $\angle BAF,\, \angle BAM$ phụ nhau nên cosin góc này bằng sin góc kia. Tương tự với các góc $\angle CAE,\, \angle CAM$.

Vậy điều cần chứng minh trở thành $$\frac{\sin\angle BAM}{\sin\angle CAM}=\frac{AK}{AH}.$$

Mặt khác, theo định lý Sin, diện tích tam giác $BAM$ bằng $\frac{1}{2}\cdot AB\cdot AM\cdot\sin \angle BAM$, diện tích tam giác $CAM$ bằng $\frac{1}{2}\cdot AC\cdot AM\cdot\sin \angle CAM$. Hơn nữa hai tam giác này có diện tích bằng nhau nên

$$AB\cdot\sin\angle BAM=AC\cdot\sin\angle CAM\Rightarrow \frac{\sin\angle BAM}{\sin\angle CAM}=\frac{AC}{AB}.$$

Vậy ta chỉ cần phải chứng minh $AH\cdot AC=AK\cdot AB$, mà điều này lại đúng do tứ giác $BKHC$ nội tiếp. 

P/s: $\downarrow$ - Cảm ơn bạn, mình post bài này lúc nửa đêm :-D




#684085 Đề thi vào 10 chuyên tỉnh Hậu Giang 2017-2018

Gửi bởi IHateMath trong 11-06-2017 - 15:14

Câu 1a)

Ta có: $(a+1)(b+1)\geq 64$

$\Leftrightarrow ab+a+b+1\geq 64$

$\Leftrightarrow ab+a+b\geq 63$(1)

Mặt khác,

$(a+b)^{2}\geq 4ab(dễ dàng chứng minh)$

$\Rightarrow \frac{1}{4}(a+b)^{2}\geq ab$(2)

$(1)\wedge(2)\Rightarrow \frac{1}{4}(a+b)^{2}+a+b\geq ab+a+b\geq 63$

$\Rightarrow \frac{1}{4}(a+b)^{2}+a+b-63\geq 0$

Đặt a+b=x(x>0),bpt trở thành

$\frac{1}{4}x^{2}+x-63\geq 0$

Giải bpt trên , ta được:

$\left\{\begin{matrix}X\leq -18(L) & \\X\geq 14(N) & \end{matrix}\right.$

$\Rightarrow Min(a+b)=14.Dấu bằng xảy ra \Leftrightarrow$

$\left\{\begin{matrix}(a+1)(b+1)=64 & \\a=b & \end{matrix}\right.$

$\Leftrightarrow a=b=7$

Vậy Min(a+b)=14 <=>a=b=7

Đâu cần phức tạp thế nhỉ?

Ta có $a+b+2=(a+1)+(b+1)\geq 2\sqrt{(a+1)(b+1)}=16\Rightarrow a+b\geq 14$. Dấu "=" xảy ra khi và chỉ khi $a=b=7$.




#684047 Đề thi vào 10 chuyên toán TP Hà Nội 2017-2018

Gửi bởi IHateMath trong 11-06-2017 - 10:42

Ai chém bài 5 đi. =))

Okay =)).

Giả sử tập $S$ có $n$ phần tử là $x_1<x_2<\dots <x_n$. Ta có $30x_n>30(x_n-x_{n-1})\geq x_nx_{n-1}\Rightarrow x_{n-1}<30.$. Vậy, từ giả thiết ta có thể suy ra $$x_{i+1}\geq \frac{30x_i}{30-x_i}.\quad (i=1,2,\dots ,n-1)$$

Từ đây kéo theo một loạt các bất đẳng thức sau

$$15>x_{n-2},$$

$$10>x_{n-3},$$

$$7\geq x_{n-4},$$

$$5\geq x_{n-5}.$$

Từ bất đẳng thức cuối ta suy ra $n-5\leq 5$, vì nếu giả sử $n-5=6$ thì ta có $1\leq x_1<x_2<\dots <x_6\leq 5$, trong khi các $x_i$ nguyên, điều này vô lí.

Vậy $n\leq 10$.

Ta chứng tỏ rằng $|S|$ có thể bằng $10$ bằng cách chỉ ra tập hợp sau

$$\{1,2,3,4,5,6,8,11,19,69\}.$$

Vậy số phần tử tối đa có thể của $S$ là $10$.




#683583 Đề thi toán chuyên - chuyên KHTN ĐHQG HÀ Nội vòng 2 2017

Gửi bởi IHateMath trong 07-06-2017 - 21:48

           ĐẠI HỌC QUỐC GIA HÀ NỘI                                                                                            ĐỀ THI TUYỂN SINH VÀO LỚP 10

TRƯỜNG ĐẠI HỌC KHOA HỌC TỰ NHIÊN                                                                       TRƯỜNG THPT CHUYÊN KHTN NĂM 2017

    ________________________________                                                                              _________________________________

                     $\boxed{\textbf{   Đề chính thức   }}$

MÔN THI: TOÁN (Vòng II)

Thời gian làm bài: 150 phút (không kể thời gian phát đề)

 

Câu I. (3,5 điểm)

1) Giải hệ phương trình

$\left\{\begin{matrix} x+y=\sqrt{x+3y} & & \\ x^2+y^2+xy=3 & & \end{matrix}\right.$

2) Với $a,\ b$ là những số thực dương thỏa mãn $ab+a+b=1$, chứng minh rằng

$$\frac{a}{1+a^2}+\frac{b}{1+b^2}=\frac{1+ab}{\sqrt{2(1+a^2)(1+b^2)}}.$$

 

Câu II. (2,5 điểm)

1) Giả sử $p,\ q$ là hai số nguyên tố thỏa mãn đẳng thức $$p(p-1)=q(q^2-1).\quad (*)$$

a. Chứng minh rằng tồn tại số nguyên dương $k$ sao cho $p-1=kq,\ q^2-1=kp$.

b. Tìm tất cả các số nguyên tố $p,\ q$ thỏa mãn đẳng thức $(*)$.

2) Với $a,\ b,\ c$ là các số thực dương thỏa mãn $ab+bc+ca+abc=2$, tìm giá trị lớn nhất của biểu thức $$M=\frac{a+1}{a^2+2a+2}+\frac{b+1}{b^2+2b+2}+\frac{c+1}{c^2+2c+2}.$$

 

Câu III (3 điểm)

Cho tam giác $ABC$ nhọn với $AB<AC$. $E,\  F$ lần lượt là trung điểm của các cạnh $CA,\  AB$. Trung trực của đoạn thẳng $EF$ cắt $BC$ tại $D$. Giả sử có điểm $P$ nằm trong $\widehat{EAF}$ và nằm ngoài tam giác $AEF$ sao cho $\widehat{PEC}=\widehat{DEF}$ và $\widehat{PFB}=\widehat{DFE}$. $PA$ cắt đường tròn ngoại tiếp tam giác $PEF$ tại $Q$ khác $P$.

1) Chứng minh rằng $\widehat{EQF}=\widehat{BAC}+\widehat{EDF}$.

2) Tiếp tuyến tại $P$ của đường tròn ngoại tiếp tam giác $PEF$ cắt các đường thẳng $CA,\ AB$ lần lượt tại $M,\ N$. Chứng minh rằng bốn điểm $C,\ M,\ B,\ N$ cùng nằm trên một đường tròn. Gọi đường tròn này là đường tròn $(K)$.

3) Chứng minh rằng đường tròn $(K)$ tiếp xúc với đường tròn ngoại tiếp tam giác $AEF$.

 

Câu IV. (1 điểm)

Cho $n$ là số nguyên dương, $n\ge 5$. Xét một đa giác lồi $n$ cạnh. Người ta muốn kẻ một số đường chéo của đa giác mà các đường chéo này chia đa giác đã cho thành đúng $k$ miền, mỗi miền là một ngũ giác lồi (hai miền bất kỳ không có điểm trong chung).

a. Chứng minh rằng ta có thể thực hiện được với $n=2018,\ k=672$.

b. Với $n=2017,\ k=672$ ta có thể thực hiện được không? Hãy giải thích.

_______________________

Cán bộ coi thi không giải thích gì thêm.

 

Nhấp vào để xem




#683580 Đề thi vào 10 chuyên tỉnh Hà Tĩnh 2017-2018

Gửi bởi IHateMath trong 07-06-2017 - 21:05

Câu $4$.

 

a. Trước hết ta có tam giác $ECD$ và tam giác $FCB$ bằng nhau, suy ra tam giác $ECF$ vuông cân tại $C$, suy ra tam giác $ENC$ vuông cân tại $N$. Mà tam giác $ABC$ cũng vuông cân nên dễ dàng suy ra tam giác $EAC$ và tam giác $NBC$ đồng dạng.

 

b. Đặt $AE=x$. Ta có $EC^2=(a+x)^2+a^2$. Vậy $S(ENC)=\frac{(a+x)^2+a^2}{2}$. Mặt khác, $S(EAC)=\frac{ax}{2}$, nên $S(ACFE)=\frac{(a+x)^2+a^2+ax}{2}$.

Ta có $$S(ACFE)=6S(ABCD)\Longleftrightarrow (a+x)^2+a^2+ax=12a^2\Longleftrightarrow 3ax+x^2=10a^2\Longleftrightarrow x=2a.$$

Khi đó $AM=\frac{ax}{a+x}=\frac{2a}{3}.$